Akademisyenler öncülüğünde matematik/fizik/bilgisayar bilimleri soru cevap platformu

Answers posted by DoganDonmez

562
answers
161
best answers
2 votes
cevaplandı 16 Ağustos 2022
Yanlış olduğu bir durum örneği vereyim. Genellikle $f(x,n)$ yerine $f_n(x)$ kullanılır. Ben de öyle
1 vote
cevaplandı 10 Ağustos 2022
Heron formülü ile de kolay çözülebiliyor. $s=\frac12(4+a+2a)=2+\frac32a$ olup, Alan$=\sqrt{s(s-a)(s
1 vote
cevaplandı 10 Ağustos 2022
Bir de Analiz kullanarak çözüm yapalım. (Geometrik çözüm daha kısa ve daha güzel)   &nbsp
1 vote
cevaplandı 19 Temmuz 2022
Önce şunu gösterelim:     Her $n\in\mathbb{Z}$ için $|\sin n|,|\sin(n+1)|,|\sin(
0 votes
cevaplandı 16 Temmuz 2022
Denklemden $p$ nin (ve $q$ nun) tek olduğunu görüyoruz. Öyleyse, Küçük Fermat Teoreminden, $2^{p-1}
1 vote
cevaplandı 16 Haziran 2022
Türev kullanarak (Ortalama Değer Teoremi ile) kolayca gösterilebilir. Zor yoldan gösterelim. Şunla
0 votes
cevaplandı 13 Haziran 2022
İstenenden daha fazlasını gösterebiliriz. Herhangi bir $\varepsilon>0$ sayısı verilsin. $|x-y|&
1 vote
cevaplandı 11 Haziran 2022
    $x^3-3x^2+1$ polinomuna, kısaca, $ P(x) $ diyelim.     $ P(-1)<
0 votes
cevaplandı 18 Mayıs 2022
Sercan ınkinden azıcık farklı bir çözüm: Mod 100 kare sayılar (0 ı saymıyorum) 1,4,9,16,25,49,64,8
0 votes
cevaplandı 18 Mayıs 2022
Önce şunları gösterelim:     $\forall x\in A\subset\mathbb{R}$ için $f(x),g(x)&g
1 vote
cevaplandı 14 Mayıs 2022
$(p,q)$ bu eşitliği sağlayan bir asal sayı çifti olsun. $p=q$ olsaydı, bu eşitlikten, $p\mid 1$ yan
0 votes
cevaplandı 12 Mayıs 2022
$\forall x\in\mathbb{R}$ için $f(x)\geq0$ olur. Bu sorunun cevabını ardarda 10 (hepsi 2. derece) de
1 vote
cevaplandı 11 Mayıs 2022
EK: Bazı işlem hatalarını düzelttim (teşekkürler alperçay) Denklemi düzenleyip: $3ab-2018a-2018b=0
0 votes
cevaplandı 8 Mayıs 2022
Sayının (10 tabanında yazılışında) soldaki 3 basamaklı kısmına $ x $ diyelim.    &nbs
0 votes
cevaplandı 8 Mayıs 2022
Daha kısa ama daha ileri düzey kavramlar kullanarak gösterilişi: (Bazı basit adımları göstermeyeceğ
0 votes
cevaplandı 7 Mayıs 2022
Bir $n$ için $(11111)_n=m^2$ olsun. Bu, $n^4+n^3+n^2+n+1=m^2$ olması demektir. ($(n^3)^2=n^6,\ 5$ d
1 vote
cevaplandı 7 Mayıs 2022
Birim çember üzerineki $A(-1,0)$ noktasını alalım. Her $m\in\mathbb{R}$ için, $A$ dan geçen, eğimi
0 votes
cevaplandı 3 Mayıs 2022
1. Cebirsel çözüm: Denklemi düzenlersek, $|z|^n=\left|{1-iz}\right|^n$ elde ederiz. $|z|,|1-iz|\g
0 votes
cevaplandı 23 Nisan 2022
OkkesDulgerci nin çözümünün (sorunun, benim gördüğüm çözümü de benzer idi) mantığı (ve belki azıcık
20,206 soru
21,731 cevap
73,293 yorum
1,894,214 kullanıcı